which transformations can be used to map a triangle with vertices A(2, 2), B(4,1), C(4, 5) to A'(-2,-2), B'(-1.-4). C'(-5, -4)?​

Answers

Answer 1

Answer:

C!

Step-by-step explanation:


Related Questions

Select the equation written in slope-intercept form that corresponds to the given slope and y-intercept. m=6 b=-2

Answers

Answer:

y = 6x - 2

Step-by-step explanation:

slope-intercept form: y = mx + b

Note that:

m = slope = 6

b = y-intercept = -2

x = (x , y)

y = (x , y)

Plug in the corresponding numbers to the corresponding variables:

y = 6x + (-2)

y  = 6x - 2

y = 6x - 2 is your answer.

~

Answer:

y = 6x-2

Step-by-step explanation:

The slope intercept equation  form of a line is

y = mx+b where m is the slope and b is the y intercept

y = 6x-2

complete the square to solve
f(x)=x^2+6x-7​

Answers


Answer: x = 1 and x = -7

Explanation:
x^2 + 6x - 7 = 0
x^2 + 6x + 3^2 = 7 + 3^2
(x + 3)^2 = 16
x + 3 = +-4 (square both sides)

x + 3 = 4
x = 1

x + 3 = -4
x = -7

April typed a 5 page report in 50 mintues. Each page had 500 words at what rate is April typing

Answers

Answer:

Amy types at a rate of 50 words per minute

Step-by-step explanation:

In this question, we are interested in calculating the rate at which April is typing.

From the question, we can deduce that she typed a 5 page report, with each page having a total of 500 words.

Now, if each page has 500 words, the total number of words in all of the pages will be 5 * 500 = 2,500 words

Now, from here, we can see that 2,500 words were typed in 50 minutes.

The number of words per minute will be ;

Total number of words/Time taken = 2500 words/50 minutes

That will give a value of 50 words per minute

To bake 12 cookies, I use 2 quarts of milk. There are 2 pints in a quart. How many pints of milk do I need to bake 3 cookies?

Answers

Answer:

1 pint

Step-by-step explanation:

You need 2 * 2 = 4 pints to make 12 cookies. Since 3 cookies is 1/4 of 12 cookies, you would need 1/4 * 4 = 1 pint of milk to make 3 cookies.

Answer:

1 pint

Step-by-step explanation:

12c=4p

divide both sides by 4

3 cookies needs 1 pint!!!!

HOPE I HELPED

PLS MARK BRAINLIEST

DESPERATELY TRYING TO LEVEL UP

       ✌ -ZYLYNN JADE ARDENNE

JUST A RANDOM GIRL WANTING TO HELP PEOPLE!

                                      PEACE!


A.) 170
B.) 300
C.)280
D.)155
Please help me

Answers

Answer:

Step-by-step explanation:

The formula for this is

∠G = 1/2(arcEH - arcHF)

We have angle G (5x - 10) and we have arcEH (195) so we have to solve for x to find the measure of arcEHF so we can add arcEH + arcHF = arcEHF

Filling in the formula with what we have:

[tex]5x-10=\frac{1}{2}(195-(8x+17))[/tex]

which simplifies down a bit to

[tex]5x-10=\frac{1}{2}(195-8x-17)[/tex] which simplifies down a bit more to

[tex]5x-10=\frac{1}{2}(178-8x)[/tex] Multiply both sides by 2 to get rid of the fraction and get:

2(5x - 10) = 178 - 8x which of course simplifies to

10x - 20 = 178 - 8x. Now add 8x to both sides and at the same time add 20 to both sides to get:

18x = 198 so

x = 11. Now we can find the measure of arcHF:

arcHF is 8x + 17, so arcHF is 8(11) + 17 which is 105°.

arcEH + arcHF = arcEHF so

195 + 105 = arcEHF so

arcEHF = 300°

A
Find the value of x. Your answer must be exact.
30
9

Answers

Tan30=9/x so it will be 9((3)^1/2)

Answer:

x = 9√3

Step-by-step explanation:

Since the figure above is a right angled triangle we can use trigonometric ratios to find x

To find x we use tan

tan∅ = opposite/ adjacent

From the question

The opposite is 9

The adjacent is x

Substitute the values into the above formula

That's

[tex] \tan(30) = \frac{9}{x} [/tex]

[tex]x \tan(30) = 9[/tex]

Divide both sides by tan 30

[tex]x = \frac{9}{ \tan(30) } [/tex]

We have the final answer as

x = 9√3

Hope this helps you

A timeline. 27 B C E to 180 C E PAX ROMANA. 44 B C E The Roman Empire was founded. 80 C E The Colosseum was built. 121 C E Hadrian's Wall was built in England to keep out enemies. 306 C E Constantine became emperor.
How many years passed between the building of the Colosseum and the building of Hadrian’s Wall?

201
121
41
36

Answers

Answer:

the answer is 41

Step-by-step explanation:

C. 41

Step-by-step explanation:

Manuel made at least one error as he found the value of this expression. Identify the step in which Manuel made his first error. After identifying the step with the first error, explain the corrected steps and find the final answer.

Answers

Answer:

Manuel made his first mistake in step 2 leading to the continuous mistakes

Final answer=185

Step-by-step explanation:

Manuel made at least one error as she found the value of this expression. 2(-20) + 3[5/4(-20)] + 5[2/5(50)] + 4(50) Step 1: 2(-20) + 3(-25) + 5(20) + 4(50) Step 2: (3 + 2)(-20 + -25) + (5 + 4)(20 + 50) Step 3: 5(-45) + 9(70) Step 4: -225 + 630 Step 5: 405 Identify the step in which Chris made her first error. After identifying the step with the first error, write the corrected steps and find the final answer.

2(-20) + 3[5/4(-20)] + 5[2/5(50)] + 4(50)

Step 1: 2(-20) + 3(-25) + 5(20) + 4(50)

Step 2: -40 - 75 + 100 +

200

Step 3: -115 + 300

Step 4: 185

Manuel made his first error in step 2 by combining two different terms into one as he has done

(3 + 2)(-20 + -25) and also (5 + 4)(20 + 50)

Step 2: (3 + 2)(-20 + -25) + (5 + 4)(20 + 50)

Step 3: 5(-45) + 9(70) Step 4: -225 + 630 Step 5: 405

He should have evaluated the terms separately as I have done above, giving us 185 as the final answer in contrast to his 405 final answer.

Una profesora compra 28 manzanas para compartir con sus estudiantes; al día siguiente revisa la cesta con las frutas y ve que se le han dañado 2/7 del total de manzanas que había comprado. Para reponer las frutas dañadas ella debe comprar

Answers

Answer:

La profesora debe comprar 8 manzanas para reponer la cesta de frutas.

Step-by-step explanation:

La profesora debe reponer la cesta de frutas por la cantidad de manzanas que se encuentran dañadas. La cantidad de manzanas dañadas es igual a las dos séptimas partes del total de manzanas. Es decir:

[tex]x = \frac{2}{7} \times (28\,manzanas)[/tex]

[tex]x = \frac{56\,manzanas}{7}[/tex]

[tex]x = 8\,manzanas[/tex]

La profesora debe comprar 8 manzanas para reponer la cesta de frutas.

28.Neethi had 8
1
4
cups of flour and 3 liters of juice with her. She decided to make cupcakes and

distribute juice for her birthday.
a) A cupcake requires 3
4
cup of flour. How many cupcakes can she make?



Answers

Answer:

2 1/3 cupcakes

Step-by-step explanation:

A cupcake can make 3 four cups of flour

In 8 fourteen cups of flour, require (14 X 4)/(3 X 8) of cupcakes

  = 56/24 = 2 1/3 cupcakes

Angles L and M are supplementary. What is the sum of
their measures?
The sum of the measures of angles L and M is

Answers

180 degree

Step-by-step explanation:

supplementary means anhke havinv sum of 180 degree

so sum to two supplemrntary angles is 180 drgree

Answer: 180

Supplementary angles always add to 180.

One way I think of it is "supplementary angles form a straight angle", and both the words "supplementary" and "straight" start with the letter "S".

In contrast, complementary angles form a corner. Both "complementary" and "corner" start with "co". By "corner", I mean a 90 degree corner.

G(x)= -\dfrac{x^2}{4} + 7g(x)=− 4 x 2 ​ +7g, left parenthesis, x, right parenthesis, equals, minus, start fraction, x, squared, divided by, 4, end fraction, plus, 7 What is the average rate of change of ggg over the interval [-2,4][−2,4]open bracket, minus, 2, comma, 4, close bracket?

Answers

Answer:

-1/2

Step-by-step explanation:

Given the function [tex]G(x)= -\dfrac{x^2}{4} + 7[/tex], the average rate of change of g(x) over the interval [-2,4], is expressed as shown below;

Rate of change of the function is expressed as g(b)-g(a)/b-a

where a - -2 and b = 4

[tex]G(4)= -\dfrac{4^2}{4} + 7\\G(4)= -\dfrac{16}{4} + 7\\G(4)= -4 + 7\\G(4) = 3\\[/tex]

[tex]G(-2) = -\dfrac{(-2)^2}{4} + 7\\G(-2)= -\dfrac{4}{4} + 7\\G(-2)= -1 + 7\\G(-2)= 6[/tex]

average rate of change of g(x) over the interval [-2,4] will be;

[tex]g'(x) = \frac{g(4)-g(-2)}{4-(-2)}\\ g'(x) = \frac{3-6}{6}\\\\g'(x) = -3/6\\g'(x) = -1/2[/tex]

In right triangle ΔABC (m∠C = 90°), point P is the intersection of the angle bisectors of the acute angles. The distance from P to the hypotenuse is equal to 4 in. Find the perimeter of △ABC if AB = 12 in.

Answers

Answer:

the perimeter of ΔABC is 32in

Step-by-step explanation:

We know that intersection point of the angle bisectors refers to the incenter of the triangle,.

Given tmthe radius of 4inch for the centre of the incircle.

One of the properties of the incircle is that the distances (d) from vertex C to the nearest touchpoints are equal and have the value

In an incircle , the distances (d) along vertex C and touchpoints have equal value and can be expressed as

d = 1/2(a +b -c)

And a, b, c represent lengths of the sides

We were given the hypotenuse (c) as 12 in, with the radius of 4inch for the

distance from the right-angle vertex C to the incircle touchpoints .

We can determine the sum a+b as

4 = (1/2)(a+b -12) .

4/(1/2)= (a+b -12)

8= (a+b -12)

20=a+b

Which is the addition of length of the two legs of the triangle.

We can determine the perimeter which is the addition of the leg lengths as well as the hypotenuse length.

perimeter = 20 in + 12 in = 32 in

Therefore, the perimeter of ΔABC is 32in


WILL GIVE BRAINLIEST!!!

Answers

Answer:

2 x^2 sqrt(13)

Step-by-step explanation:

sqrt( 52x^4)

sqrt( 4*13 * x^2 * x^2)

We know that sqrt(ab) = sqrt(a) sqrt(b)

sqrt( 4)*sqrt(13) *sqrt( x^2) *sqrt( x^2)

2 sqrt(13) x*x

2 x^2 sqrt(13)

52|2

26|2

13|13

1

[tex]\sqrt{52x^4}=\sqrt{2^2\cdot13\cdot(x^2)^2}=2x^2\sqrt{13}[/tex]

Jon wants to buy carpet to cover his whole living room, except for the tiled floor. The tiled floor
is
oft by 3 1 ft. Find the area the carpet needs to cover. •!
12
4
5
tiled floor
Entar

Answers

Answer:

321/8 ft^2

Step-by-step explanation:

The whole area is 49/4 × 5 = 245/4 ft^2

The area of tiled floor is 13/2 × 13/4 = 169/8

245/4 - 169/8 we need to equalise the bottom of fraction to subtract so

2/2 × 245/4 = 490/8

490/8 - 169/8 = 321/8 ft^2

Help please!!! Thank you

Answers

Answer:

2y+6x=180

Step-by-step explanation:

Because we know that side lengths BD, DC, and AD are all congruent, we can conclude that triangles BDA and CDA are congruent because they have at least two congruent sides. Since these triangles are both 45-45-90 triangles, angle C is equal to 45 degrees, or 3x. 45/3 is 15, so x=15. Angle B is equal to 45 degrees, or y, so y=45.

From there, we plug these numbers into the equation with 2(45) + 6(15), or 90+90 = 180.

What is 25x + 67y if x = 23 and y = 36. Give explanation please! ​

Answers

Answer:

2987.

Step-by-step explanation:

25(23) + 67(36) = 575 + 2412 = 2987.

Hi there! Hopefully this helps!

------------------------------------------------------------------------------------------------------------

Answer: 2987

First we need to rewrite the equation. Since x = 23 and y = 36 the equation should look like this for easier steps:

25(23) + 67(36) = ?

~~~~~~~~~~~~~~~~~~~~~~~~~~~~~~~~~~~~~~~~~~~~~~~~~~~~~~~~~~~~~~~

Now since there numbers by other numbers in parentheses, we need to multiply them.

25 x 23 = 575.

67 x 36 = 2412.

~~~~~~~~~~~~~~~~~~~~~~~~~~~~~~~~~~~~~~~~~~~~~~~~~~~~~~~~~~~~~~~~~

Now that the equation is in its final form, we write it like this for the answer:

575 + 2412 =

2987.

explain the difference between legs and the hypotenuse of a right triangle ​

Answers

Answer:

The difference between the legs and the hypotenuse of a right triangle is that the hypotenuse will always be the longest side. Also, it will always be less than the two legs added together.

Step-by-step explanation:

Prove: The square of the sum of
two consecutive integers is odd.

Answers

[tex](2n+1)^2=4n^2+4n+1[/tex] therefore, the first blank is 1.

[tex]4n^2+4n+1=2(2n^2+2n)+1[/tex] therefore, the two other blanks are both 2.

The number in the proof ''The square of the sum of two consecutive integers is odd'' is 2 and 2.

To prove that, The square of the sum of two consecutive integers is odd.

The expression to prove is,

Let us assume that two consecutive integers are n and (n + 1).

Hence, the expression is written as,

[n + (n + 1)]² = (2n + 1)²

= (2n)² + 2 × 2n × 1 + 1²

= 4n² + 4n + 1

= 2 (2n² + 2n) + 1

= odd

Therefore, the number in the blanks are 2 and 2.

To learn more about the Number system visit:

https://brainly.com/question/17200227

#SPJ4

Can someone PLEASE help with this question? thank you

Answers

Answer:

C) 1

Step-by-step explanation:

First half:

Invert and multiply

x²/y²*y³/x²=x²y³/y²x³=y/x

Second half:

Invert and multiply

1/y*x/1=x/y

Combine

y/x*x/y=xy/xy=1

Anyone the answers? Please help

Answers

Answer:

1.) 3 shovels/7 buckets

2.)7 buckets/3shovels

Please help will give 5 stars with 1 thanks and 15 points

Answers

Answer:

mean is adding up all the numbers.

range means the difference between the Lowest and highest value.

Step-by-step explanation:

when we add the answer is 240.6.

divide it to 12..do the final answer is 20.05

range is 25.4_16.3

9.1

Answer:

Mean: 20.05

Range: 9.1

Step-by-step explanation:

To find the mean in this problem, first you add all the following numbers together then subtract by the quantity.

20.1 + 19.6 + 18.0 + 17.8 + 25.2 + 18.7 + 21.9 + 16.3 + 25.4 + 20.5 +17.8 + 19.3

= 240.6

Now, divide by the quantity which is 12 since there's 12 numbers.

240.6 divided by 12 = 20.05.

The mean is 20.05.

To find the range in the problem, you must subtract the smallest value from the largest value.

In this case, 16.3 is the smallest value and 25.4 is the largest.

25.4 - 16.3 = 9.1

9.1 is the range.

Hope this helps you!

The drama club is selling tickets to its play. An adult ticket costs $15 and a student ticket costs $11. The auditorium will seat 300 ticket-holders. The drama club wants to collect at least $3630 from ticket sales.

Answers

Answer:

83 adult tickets and 217 student tickets.

Step-by-step explanation:

Let number of adult tickets sold = [tex]x[/tex]

Given that total number of tickets = 300

So, number of student tickets = 300 - [tex]x[/tex]

Cost of adult ticket = $15

Cost of student ticket = $11

Total collection from adult tickets = $[tex]15x[/tex]

Total collection from student tickets =  [tex](300-x)\times 11 = 3300-11x[/tex]

Given that overall collection = $3630

[tex]15x+(3300-11x) = 3630\\\Rightarrow 15x-11x=3630-3300\\\Rightarrow 4x = 330\\\Rightarrow x = 82.5[/tex]

So, for atleast $3630 collection, there should be 83 adult tickets and (300-83 = 217 student tickets.

Now , collection = $3632

-3 = 7 - BLANK pls tell me what blank is

Answers

Answer:

10

Step-by-step explanation:

-3 = 7 - x

Add x to both sides

x -3 = 7 - x +x

x - 3 = 7

Now, add 3 to both sides

x - 3 + 3 = 7 + 3

x = 10

Answer:

[tex]\boxed{10}[/tex]

Step-by-step explanation:

[tex]-3=7- \sf BLANK[/tex]

[tex]\sf Subtract \ 7 \ from \ sides.[/tex]

[tex]-3-7=-7+7- \sf BLANK[/tex]

[tex]-10=- \sf BLANK[/tex]

[tex]\sf Multiply \ both \ sides \ by \ -1.[/tex]

[tex]-10(-1)=(-1)- \sf BLANK[/tex]

[tex]10= \sf BLANK[/tex]

Convert 1.2 radians to an
angle in degrees​

Answers

Answer:

Below

Step-by-step explanation:

Let x be the measure in degrees

● x/180 = 1.2/Pi

Cross multiply the values

● x*Pi = 180×1.2

● x = 216/Pi

Take Pi = 3.14

● x = 68.78 wich is approximatively 69°

Marcus played 3 different activities this week after school. The time he spent playing is described below. Swam forof an hour Played soccer for 9/10 2/3 2/4 of an hour Jogged forof an hour Which statement correctly compares the times of 2 of his activities?

Answers

Answer:

A

Step-by-step explanation:

We can find a common denominator for all of these fractions, compute, and then compare the values to find the right answer.

Answer it answer it answer it.

Answers

Answer:

Option C. P = 3/q

Step-by-step explanation:

To know the the correct answer to the question, do the following:

Let us assume a certain number for P say 2 and 3, and then, find the corresponding value for q in each case to see which will give a decreased value for q.

Option A

When P = 2, q =.?

P = 3q

2 = 3q

Divide both side by 3

q = 2/3

When P = 3, q =.?

P = 3q

3 = 3q

Divide both side 3

q = 3/3

q = 1

From the above illustration, we can see that as P increase, q also increase.

Option B

When P = 2, q =.?

P – 3 = q

2 – 3 = q

q = – 1

When P = 3, q =.?

P – 3 = q

3 – 3 = q

q = 0

From the above illustration, we can see that as P increase, q also increase.

Option C

When P = 2, q =.?

P = 3/q

2 = 3/q

Cross multiply

2 × q = 3

Divide both side by 2

q = 3/2

q = 1.5

When P = 3, q =.?

P = 3/q

3 = 3/q

Cross multiply

3 × q = 3

Divide both side by 3

q = 3/3

q = 1

From the above illustration, we can see that as P increase, q decreases.

Option D.

When P = 2, q =.?

1/p = 3/q

1/2 = 3/q

Cross multiply

1 × q = 2 × 3

q = 6

When P = 3, q =.?

1/p = 3/q

1/3 = 3/q

Cross multiply

1 × q = 3 × 3

q = 9

From the above illustration, we can see that as P increase, q also increase.

Now, haven done the above, only option C gives a decreased value for q as the value of P increases.

c  

this before

Step-by-step explanation:

Solve.

-7(2z + 4) = 21

Answers

Answer:

-7/2

Step-by-step explanation:

cuz thats right

-14z - 28 = 21
-14z = 49
z = -7/2

f(x) = x2. What is g(x)?

Answers

Answer:

g(x)=3x(superscript)2

Answer:

g(x) = 3x².

Step-by-step explanation:

In the diagram, we see that the vertex has not been shifted from the origin. The only thing that happened to the graph of f(x) was that it was vertically stretched to become g(x).

Where x = 1, f(x) = 1. Where x = 1, g(x) = 3. That means that the graph of f(x) was multiplied by 3.

So, g(x) = 3x².

Hope this helps!

GIVING BRAINLIEST TO THE FIRST PERSON TO ANSWER!

Three stores have the same tablet computer on sale. The regular price of the tablet is $150. Store A is offering the tablet on sale at 15% off the regular price. Store B is offering a $25 coupon to be deducted from the regular price. Store C is offering a rebate of $20.00 to purchasers. Store D has the tablet on sale for $120.00. Which store is offering the tablet at the lowest cost?

A. Store A

B. Store B

C. Store C

D. Store D

Show ALL work please! <3

Answers

Answer: Store D

Step-by-step explanation:

Store A - 15% off:

$150 times 15/100 = 22.5

$150 - $22.50 = $127.50 so the price at store A is $127.50

Store B - $25 coupon

$150 - $25 = $125 so the price at store B is $125

Store C - $20 rebate

$150 - $20 = $130 so the price at store B is $130

Store D - $120

$120 is the lowest so the answer is Store D

Answer:

[tex]\Large \boxed{\mathrm{Store \ D}}[/tex]

Step-by-step explanation:

Store A is offering the tablet on sale at 15% off the regular price.

150 × (1 - 15%) = 127.5

Store B is offering a $25 coupon to be deducted from the regular price.

150 - 25 = 125

Store C is offering a rebate of $20.00 to purchasers.

150 - 20 = 130

Store D has the tablet on sale for $120.00.

Store D is offering the tablet at the lowest cost.

Other Questions
Disturbed Corp. needs to raise $57 million to fund a new project. The company will sell shares at a price of $23.70 in a general cash offer and the company's underwriters will charge a spread of 7.5 percent. The direct flotation costs associated with the issue are $725,000 and the indirect costs are $445,000. How many shares need to be sold? I NEED HELP!!! What was the CIAs response to a potential military coup against Diem? A. They would launch a counter-attack against any forces attempting a coup B. They would not interfere in a coup C. They would initiate the coup D. They would protect Diem at all costs Salaries of 42 college graduates who took a statistics course in college have a mean, , of . Assuming a standard deviation, , of $, construct a % confidence interval for estimating the population mean . Let X denote the day she gets enrolled in her first class and let Y denote the day she gets enrolled in both the classes. What is the distribution of X What does y equal -3=15+4y What complications might arise from genetic screens targeting an organ that differentiates late in development? A bin contains seven red chips, nine green chips, three yellow chips, and six blue chips. Find each probability. drawing a yellow chip, replacing it, and choosing a blue chip. Use distributive property to evaluate the expression 5(4/1/5) Why HPE education is made compulsory in the secondary level of education in nepal?Give reason. The basic unit in which data are stored in an accounting system is called an __________. These storage units should be so constructed as to readily receive money measurements of the __________ or ___________ in the items for which they are established. How has your learning inside and outside of the classroom changed your perspective about experiences you have in school and life in general On February 3, 1969, New York lawyer and businessman _______________ was appointed the Beatles' business manager, as John was impressed by what the man had done financially for the Rolling Stones. Which of the following is NOT a product of the electrolysis of NaCl? A. Chlorine gas B. None of these C. Hydrogen gas D. Oxygen gas Max believes that the sales of coffee at his coffee shop depend upon the weather. He has taken a sample of 5 days. Below you are given the results of the sample. Cups of Coffee Sold Temperature 350 50 200 60 210 70 100 80 60 90 40 100 A. Which variable is the dependent variable? B. Compute the least squares estimated line. C. Compute the correlation coefficient between temperature and the sales of coffee. D. Predict sales of a 90 degree day. find the area of square whose side is 2.5 cm How many solutions does the nonlinear system of equations graphed belowhave?y10+-1010-10A. OneB. Two0OC. FourOD. Zero Choose the word showing the correct stress/accent. The syllable written in uppercase letters is the accented syllable. The students (survey) the island where they will conduct their case studies. a. survey b. SURvey c. surVEY d. SURVEY 88 feet/second = 60 miles/hour. How many feet per second is 1 mile/hour? (Hint: divide both sides of the equationby the same amount.)Round to the nearest thousandth.One mile per hour is equivalent toao feet/second Please answer this question now If the sphere shown above has a radius of 17 units, then what is the approximate volume of the sphere? A. 385.33 cubic units B. 4,913 cubic units C. 6,550.67 cubic units D. 3,275.34 cubic units